Inscription / Connexion Nouveau Sujet
Niveau maths spé
Partager :

Complément orthogonal du complément orthogonal

Posté par
Serbiwni
26-06-21 à 12:30

Bonjour à tous,
Soit \langle , \rangle un produit scalaire dans un espace vectoriel V sur un corps K. Je voulais savoir ce que l'on pouvait dire de (W^\perp)^\perp avec W un sous-espace vectoriel de V.

J'ai entendu dire que si le corps est \mathbb R ou \mathbb C, et si W est un espace de Hilbert  (je ne sais pas encore ce que s'est) alors (W^\perp)^\perp = \overline{W}.
J'ai également lu que si le sous-espace vectoriel W est de dimension finie alors (W^\perp)^\perp = W.
Que peut-on dire dans le cas général ? Quand K=\mathbb R, a-t-on (W^\perp)^\perp \subseteq W ou plutôt (W^\perp)^\perp \supseteq W pour tout \mathbb R-espace vectoriel W ?

On m'a dit qu'en général la première inclusion est fausse et que la deuxième est toujours vraie. On m'a aussi indiqué de considérer un R-espace vectoriel non fermé pour voir ce qui ne marche pas dans (W^\perp)^\perp = \overline{W}.

Quelqu'un peut-il m'éclairer sur cette relation entre   (W^\perp)^\perp et W ?

Posté par
WilliamM007
re : Complément orthogonal du complément orthogonal 26-06-21 à 14:00

Bonjour.

Citation :
si W est un espace de Hilbert  (je ne sais pas encore ce que s'est)

Ce serait plutôt V, l'espace de Hilbert. Un espace de Hilbert est un espace muni d'un produit scalaire, complet (sous-entendu, pour la norme induite par ce produit scalaire). Par exemple tout espace vectoriel de dimension finie est de Hilbert.

Citation :
Que peut-on dire dans le cas général ? Quand K=\mathbb R, a-t-on (W^\perp)^\perp \subseteq W ou plutôt (W^\perp)^\perp \supseteq W pour tout \mathbb R-espace vectoriel W ?

Je ne suis pas sûr d'avoir bien compris ta question. Tu l'as dit toi-même : (W^\perp)^\perp=\overline W, donc on a clairement W\subset\overline W=(W^\perp)^\perp. Cette inclusion est assez facile à voir. En effet, soit x\in\overline W. Alors il existe une suite (x_n)_{n\in\N} d'éléments de W convergeant vers x. Soient n\in\N et y\in W^\perp. Puisque x_n\in W, on a par définition que \langle x_n,y\rangle=0. Par continuité du produit scalaire, lorsque n\to+\infty, on obtient \langle x,y\rangle=0. On a donc montré :
\forall x\in W,\quad\forall y\in W^\perp,\quad\langle x,y\rangle=0,
ce qui est exactement dire que x\in(W^\perp)^\perp, d'où l'inclusion.

Citation :
On m'a dit qu'en général la première inclusion est fausse

Effectivement, puisque (W^\perp)^\perp=\overline W, on a (W^\perp)^\perp\subset W ssi W=\overline W, donc tout sous-espace vectoriel non fermé fournit un contre-exemple.

En fait, sans même dire que (W^\perp)^\perp=\overline W, il est facile de voir que si A est un sous-ensemble de V (même pas nécessairement un espace vectoriel), alors A^\perp est un sous-espace vectoriel fermé de V. En particulier, (W^\perp)^\perp doit donc être fermé, donc ne peut être égal à W si ce dernier n'est pas fermé. À partir de ce constat, il est donc clair qu'il existe un espace vectoriel fermé H, contenant W, tel que (W^\perp)^\perp=H. On sait en fait que H=\overline W, ce qui fait l'objet d'un théorème, peut-être un tout petit peu moins évident à démontrer que ce que j'ai raconté plus haut, mais pas très difficile non plus.

Juste une précision. Tout espace vectoriel de dimension finie est fermé, d'où la relation (W^\perp)^\perp=W valide en dimension finie. Mais ce n'est qu'une réécriture du fait que (W^\perp)^\perp=\overline W et \overline W=W.

Posté par
Serbiwni
re : Complément orthogonal du complément orthogonal 26-06-21 à 20:56

WilliamM007 Merci beaucoup, c'est vraiment très clair à présent. Je voulais juste savoir si le fait que W^\perp=\{0\} changerait quelque chose à la non-validité de (W^\perp)^\perp \subset W en dimension infinie. Si j'ai bien compris ça ne devrait pas être forcément le cas car cela n'arrive que si W = \overline{W} ce qui n'a rien à voir avec W^\perp

Posté par
WilliamM007
re : Complément orthogonal du complément orthogonal 27-06-21 à 13:28

On sait que W^\perp=\{0\} ssi W est dense dans V, soit \overline W=V.

Si W^\perp=\{0\}, alors (W^\perp)^\perp=\{0\}^\perp=V, donc (W^\perp)^\perp\subset W ssi W=V.



Vous devez être membre accéder à ce service...

Pas encore inscrit ?

1 compte par personne, multi-compte interdit !

Ou identifiez-vous :


Rester sur la page

Inscription gratuite

Fiches en rapport

parmi 1675 fiches de maths

Désolé, votre version d'Internet Explorer est plus que périmée ! Merci de le mettre à jour ou de télécharger Firefox ou Google Chrome pour utiliser le site. Votre ordinateur vous remerciera !